1answer.
Ask question
Login Signup
Ask question
All categories
  • English
  • Mathematics
  • Social Studies
  • Business
  • History
  • Health
  • Geography
  • Biology
  • Physics
  • Chemistry
  • Computers and Technology
  • Arts
  • World Languages
  • Spanish
  • French
  • German
  • Advanced Placement (AP)
  • SAT
  • Medicine
  • Law
  • Engineering
ANTONII [103]
3 years ago
13

(LC)Light fixtures and placement that create shadows on the set, that obscure or completely hide action in certain areas of the

set, or that change as the main character’s emotional state changes are all ways that lighting can be used to heighten the drama and suspense in dramatic films. True/False
Physics
1 answer:
Sedaia [141]3 years ago
5 0

The correct answer is true.

It is true that light fixtures and placement that create shadows on the set, that obscure or completely hide action in certain areas of the set, or that change as the main character’s emotional state changes are all ways that lighting can be used to heighten the drama and suspense in dramatic films.

Lighting plays an important role in film making because it can create scenes that enhance the de drama of the moment or the right mood that the director wants to share. Lighting in the film is an art because the basic principle is that the scene needs to look natural. From that principle, filmmakers and light specialist cand create many kinds of dramatic or jubilation moments if they know how to apply light principles to each scene.

You might be interested in
To sterilize a 53.0 g glass baby bottle, we must raise its temperature from 25.0°C to 92.0°C. How much heat transfer (in J) is r
Stella [2.4K]

The heat energy required is 2983 J

Explanation:

When an amount of energy Q is supplied to a certain sample of substance of mass m, the temperature of the substance increases by an amount of \Delta T according to the equation :

Q=mC_s \Delta T

where

Q is the heat supplied

m is the mass of the object

C_s is the specific heat capacity of the substance

\Delta T is the change in temperature

For the glass bottle in this problem:

m=53.0 g

\Delta T = 92.0 - 25.0 =67.0^{\circ}C is the change in temperature

C_s = 0.84 J/g^{\circ}C is the specific heat of glass

Substituting into the equation, we find

Q=(53.0)(0.84)(67)=2983 J

Learn more about specific heat capacity:

brainly.com/question/3032746

brainly.com/question/4759369

#LearnwithBrainly

8 0
3 years ago
Blake stands in a canoe in the middle of a lake. The canoe is stationary. Blake holds an anchor mass of 15 kg, then throws it we
Inessa05 [86]

The velocity of the canoe is  1.7 m/s.

<h3>What is momentum?</h3>

Momentum in physics is the products of mass and velocity. Now we have to find momentum with the formula; p = mv

a) Initial momentum = (15)8 m/s + 135 = 255 Kgms-1

b) Since momentum is conserved, the total momentum after throwing the anchor is still 255 Kgms-1

c) The final velocity of the boat is obtained from;

255 Kgms-1 = (15Kg + 135 Kg) v

v = 255 Kgms-1/(15Kg + 135 Kg)

v = 1.7 m/s

Learn more about momentum: brainly.com/question/904448

5 0
2 years ago
According to Coulomb's law, the electrical force between two charged objects:A.is zero if they are opposite charges.B.increases
kondor19780726 [428]

Answer and explanation:

A correct option is an option (B).

The electrical force between two charges is given as,

F=\frac{1}{4\pi\epsilon_0}\frac{q_1q_1}{r^2}

The electrical force is directly proportional to the product of two charges. Thus Force will depend on two charges irrespective of their signs.

Option (A) is incorrect because if charges are opposite, the value of force will not be zero. It will be -ve.

Option (C) is incorrect because the force is directly proportional to the product of charges, it depends on the amount of charge.

Option (D) is also incorrect because the force in inversly proportional to the distance between two charges. Thus, if the distance between two charges is increased, the force between two charges will decrease.

Concllusion:

The correct option is option (B).

7 0
11 months ago
Alan leaves Los Angeles at 8:00 A.M. to drive to San Francisco 400 mi away. He manages to travel at a steady 50 mph in spite of
Fudgin [204]

Answer:

a) Beth will reach before Alan

b)Beth has to wait 20 min for Alan to arrive

Explanation:

let 'd' be distance b/w Los Angeles and San  Francisco i.e 400 mi

considering ,

Alan's speed v_A=50mph

Beth's speed v_B=60mph

->For Alan:

The time required t_A= d/v_A= 400/50 => 8h

-> For beth:

The time required t_B=\frac{d}{v_B} =\frac{400}{60} =>6\frac{2}{3} h => 6h 40m

Alan will reach at 8:00 a.m +8h = 4:00p.m.

Beth will reach at 9:00 a.m +6h 40m= 3:40p.m.

a) Beth will reach before Alan

b)Beth has to wait 20 min for Alan to arrive

3 0
3 years ago
Ben helped his parents load boxes and furniture onto the moving truck. He easily carried several boxes out of the house and up a
NikAS [45]
D) because the refrigerator was much heavier than the other objects
5 0
3 years ago
Read 2 more answers
Other questions:
  • An electromagnetic wave is propagating towards the west. At a certain moment the direction of the magnetic field vector associat
    8·1 answer
  • Which best describes a graph of voltage over time for DC current?
    15·1 answer
  • A person pulls a crate of mass M = 63 kg a distance 40.0 m along a horizontal floor by a constant force FP = 130 N, which acts a
    6·1 answer
  • A CD spins at a constant angular velocity of 5.0 revolutions per second clockwise.
    10·1 answer
  • Which of the following accurately describes circuits? A. In a parallel circuit, the same amount of current flows through each pa
    15·2 answers
  • why does a car slide when it rolls over an icy patch on a road? A. cars have more momentum on icy surfaces. B. there is less fri
    9·2 answers
  • 2.0 mol of monatomic gas A initially has 5000 J of thermal energy. It interacts with 3.0 mol of monatomic gas B, which initially
    7·1 answer
  • A closed container initially holds 50 monatomic Aparticles that have a combined energy of 480 units. After 100 monatomic B parti
    13·1 answer
  • Find the Y component of 35m/s at 57 degrees from the X-axis.
    12·1 answer
  • This diagram shows a heating curve for water.
    13·1 answer
Add answer
Login
Not registered? Fast signup
Signup
Login Signup
Ask question!